User avatar
 
ohthatpatrick
Thanks Received: 3808
Atticus Finch
Atticus Finch
 
Posts: 4661
Joined: April 01st, 2011
 
 
 

Re: Q6 - Beck: Our computer program estimates

by ohthatpatrick Fri Dec 31, 1999 8:00 pm

What does the Question Stem tell us?
Flaw

Break down the Stimulus:
Conclusion: We can be confident that the program's figures are accurate.
Evidence: The program's figures are consistent from week to week.

Any prephrase?
This seems to be a term shift gap between something being "consistent" and something being "accurate". I could consistently call you Peter, even though you're name is Paul. So consistency doesn't prove accuracy. The author assumes "if it's consistent, it's accurate". An objection to her argument would be to say "something can be consistent, but INACCURATE".

Correct answer:
E

Answer choice analysis:
A) The author doesn't need to rank consistency over accuracy. She only needs to convince us that if something is consistent, then it's accurate.

B) The "accuracy" in question refers to the program's weekly estimates. So it's irrelevant to this debate whether the program happens to be accurate or inaccurate at other tasks. The author is only vouching for its accuracy concerning its consistent weekly estimates.

C) The author does NOT assume this. She is thinking that consistency implies accuracy. This answer says the opposite, that consistency is unrelated to accuracy (even if = irrelevance).

D) The author regards "consistency" as the sole criterion for judging the programs "accuracy".

E) Looks good. "Fails to consider" = "Would this weaken?" Yes it would. If the program is churning out consistently INACCURATE output, then the author's conclusion is dead wrong.

Takeaway/Pattern: The argument core contained a simple language shift that we likely notice, but the answer choices can still make things tricky. Always beware STRONG ideas, like "SOLE criteriorn", and NEW COMPARISONS, "consistency is MORE IMPORTANT than accuracy". The most famous out of scope word is "other". We're evaluating an argument about how accurate this program's estimates are, and (B) wants to talk about how accurate it is at OTHER stuff. (C) is trying to provide a necessary assumption. (E) is trying to provide a weaken idea. Judge them accordingly.

#officialexplanation
 
carefreeai36
Thanks Received: 0
Forum Guests
 
Posts: 4
Joined: September 13th, 2011
 
 
 

Q6 - Beck: Our computer program estimates

by carefreeai36 Mon Sep 26, 2011 8:25 am

I can see [E] is the answer. but would [C] be a right choice if [E] is not there? Need help :cry:
User avatar
 
maryadkins
Thanks Received: 641
Atticus Finch
Atticus Finch
 
Posts: 1261
Joined: March 23rd, 2011
 
This post thanked 1 time.
 
 

Re: Q6 - Beck: Our computer program estimates

by maryadkins Wed Sep 28, 2011 8:32 am

Our core is:

the computer program is consistent --> it's accurate

The flaw here is that consistency means accuracy. (E) works.

(A) is out of scope. More important?
(B) is also out of scope. Other tasks?
(C) Not quite, but tricky. Our assumption is If it’s C then it’s A. C --> A. This answer says it assumes -A -->C. (Also, think about it. (C) is saying it can be consistent but inaccurate. That's the opposite of what we're looking for.)
(D) is also out of scope.
 
sumukh09
Thanks Received: 139
Atticus Finch
Atticus Finch
 
Posts: 327
Joined: June 03rd, 2012
 
 
trophy
Most Thanked
trophy
First Responder
 

Re: Q6 - Beck: Our computer program estimates

by sumukh09 Sat Sep 22, 2012 12:30 am

Hey MLSAT,

I'm not sure I see the difference between answer choice C) and E).

Maybe I've misinterpreted the stimulus but is there a difference between output and estimates? Does municipal automotive use = estimates and weekly data = output? Is that why C) is wrong and E) is right? Because C) says that municipal automotive use estimates are consistent but the weekly data is inaccurate when it should say the weekly data is consistent but the municipal automotive use estimates are inaccurate?
 
ilia.medovikov
Thanks Received: 5
Forum Guests
 
Posts: 13
Joined: July 02nd, 2013
 
 
 

Re: Q6 - Beck: Our computer program estimates

by ilia.medovikov Mon Jul 08, 2013 1:39 pm

Another way to understand why answer choice (c) is wrong is to look at its contrapositive:

Answer choice (C) original statement: ~A----->C
Answer choice (C) contrapositive: ~C ------->A: If the figures that the program provides are not consistent, then the program is accurate. The author of the stimuli, on the other hand, assumes that if the figures are consistent, then they are accurate. As such, answer choice C does not describe the flow: equating consistency with accuracy.
 
timsportschuetz
Thanks Received: 46
Elle Woods
Elle Woods
 
Posts: 95
Joined: June 30th, 2013
 
 
trophy
First Responder
 

Re: Q6 - Beck: Our computer program estimates

by timsportschuetz Fri Nov 15, 2013 10:44 pm

(C) is wrong because "even if" does NOT denote conditional reasoning! Therefore, you cannot use this statement to infer any conditional logic...
 
mshinners
Thanks Received: 135
Atticus Finch
Atticus Finch
 
Posts: 367
Joined: March 17th, 2014
Location: New York City
 
 
 

Re: Q6 - Beck: Our computer program estimates

by mshinners Fri May 29, 2015 10:48 am

timsportschuetz Wrote:(C) is wrong because "even if" does NOT denote conditional reasoning! Therefore, you cannot use this statement to infer any conditional logic...


Correct in the analysis of "even if", but we don't need an answer that has conditional logic here - that's not the reason (C) is wrong.. In fact, (E) doesn't really have conditional logic, either, for the same reason ("could produce"). Using conditional logic to understand it is a shortcut.

The difference between (C) and (E) is ONLY in the first couple words of the answer choice - "takes for granted" vs. "fails to consider". Outside of that, they're the same answer (we can have consistent and inaccurate results). It's important to recognize these answer choice constructs in ID the Flaw questions because they change how we analyze the answer.

So let's look at the two:
(C) - This states the argument assumes that you can be both consistent and inaccurate. But does the argument assume that? Nope! In fact, quite the opposite - it assumes that consistent results will always be accurate.

(E) - Does the argument, then, overlook that you could have consistent results that are also inaccurate? Yep - that's a possibility that the argument doesn't consider. If consistent and inaccurate results are possible, then making that jump between consistent and accurate isn't cool.

Another way to look at it - a lot of ID the Flaw questions will start with either "assumes" or "overlooks" (or synonymous phrases). If it starts with "assumes", then the answer choice is attempting to point out the assumption in the argument - and if we state the assumption as fact, then that helps the argument. So if my answer choice in an ID the Flaw question starts with "assumes", if I put the rest of the answer in the argument, it should serve as a strengthener. On the flipside, if I make an argument and I overlook something, that hurts my argument. So if I look at the rest of an answer that comes after "overlooks", it should serve as a weakener.

Using that here, (C) starts with "assumes" but gives me a piece of information that weakens the argument - my results might be both consistent and inaccurate.

(E) gives me the same thing, but it frames it with "overlooks" - and that matches up! I need whatever my argument overlooks to weaken my argument, and here, it does.
 
stacksdoe
Thanks Received: 0
Forum Guests
 
Posts: 54
Joined: August 19th, 2012
 
 
 

Re: Q6 - Beck: Our computer program estimates

by stacksdoe Sat Sep 05, 2015 11:59 am

Both are accurate description though. I don't think focusing on conditionality is a bad thing, in fact, conditionality is the most prevalent concept on the entire lsat (less so on RC). Thus, developing and understanding and how to work with conditional statement will only help and not hurt your performance.
 
ahm
Thanks Received: 0
Vinny Gambini
Vinny Gambini
 
Posts: 3
Joined: June 20th, 2015
 
 
 

Re: Q6 - Beck: Our computer program estimates

by ahm Sun Feb 14, 2016 2:49 pm

If C had said "Fails to consider" as opposed to "takes for granted" would that have made it a viable answer? I think it would have, and I'd like to make sure I'm understanding it correctly.